how would i go about this question?

How Would I Go About This Question?

Answers

Answer 1

Answer:

  x = 48.40'

  y = 33.01'

Step-by-step explanation:

These are all segments of a right triangle, so the Pythagorean theorem applies.

  x^2 = 46.07^2 +14.82^2 = 2342.0773

  x = √2342.0773 ≈ 48.40

__

  x^2 = 35.39^2 +y^2

  y^2 = 2342.0773 -35.39^2 = 1089.6252

  y = √1089.6252 ≈ 33.01

Units are feet for both numbers:

  x = 48.40'

  y = 33.01'

_____

You always start any question by reading the whole question, identifying the (relevant) given information, and understanding what the question is asking for. You then make an assessment of what you know about the relationships between the given information and what is asked. Finally, you develop a strategy for getting from what you know to what you need to know.

Here, you're given segment lengths of right triangles, and you're asked for other segment lengths. This is not a trig problem--no angles are involved. It is a straight Pythagorean theorem problem. To make use of that theorem, or any formula, you need to have only one unknown. So, you can't start by solving the bottom triangle; you have to start with the upper one where there is only the unknown side x.

After you find x, then you have two known sides in the bottom triangle, so you can find y.

Here, x is an intermediate value in the computation of y. You do NOT use the rounded answer to the question when you're computing y. Rather, you use the full calculator precision for x, so that y can have best accuracy. Only at the end do you round to hundredths.


Related Questions

What’s the correct answer for this? Select two answer choices

Answers

Answer:

D and F

Step-by-step explanation:

In the attached file

Sloane kicked a soccer ball at a speed of 48 feet per second. If the ball never leaves the ground, then it can be represented by the function H(t) = −16t2 + 48t. Determine the time the ball traveled. t = 0.33 seconds t = 3 seconds t = 16 seconds t = 48 seconds

Answers

Answer:

t = 3 seconds

Step-by-step explanation:

I took the test and i got it right.

The time the given ball traveled for is required.

The time the ball traveled was t = 3 seconds

Equation of motion

The given equation is [tex]H(t)=-16t^2+48t[/tex]

The ball never leaves the ground so the height of the ball is 0.

[tex]0=-16t^2+48t[/tex]

Solving the equation we get

[tex]-48t=-16t^2[/tex]

Dividing the equation by [tex]t[/tex] on both sides

[tex]\Rightarrow 48=16t[/tex]

Dividing the equation by 16 on both sides

[tex]\Rightarrow t=\dfrac{48}{16}\\\Rightarrow t=3\ \text{s}[/tex]

Learn more about equation of motion:

https://brainly.com/question/25951773

rewrite the following equation in logarithmic form and exponential form

Answers

Answer:The answer for the first one is log2(.25)=-2

The answer for the second one is 8^3=512

Step-by-step explanation:

Write the following statement as a unit rate: 18 laps in 6 minutes

Answers

Answer:

3 laps per minute

Step-by-step explanation:

Take the number of laps and divide by the minutes

18/6 = 3 laps per minute

Answer:

3 laps per min

Step-by-step explanation: divide 18 and 6


When you add two rational numbers, each number
can be written as a ?

Answers

a Rational # , because when you add two rational #’s each number can be written as a rational #

ssume that the readings at freezing on a batch of thermometers are normally distributed with a mean of 0°C and a standard deviation of 1.00°C. A single thermometer is randomly selected and tested. Find the probability of obtaining a reading between -1.94°C and -1.5°C.

Answers

Answer:

[tex]P(-1.94<X<-1.5)=P(\frac{-1.94-\mu}{\sigma}<\frac{X-\mu}{\sigma}<\frac{-1.5-\mu}{\sigma})=P(\frac{-1.94-0}{1}<Z<\frac{-1.5-0}{1})=P(-1.94<z<-1.5)[/tex]

And we can find this probability with this difference:

[tex]P(-1.94<z<-1.5)=P(z<-1.5)-P(z<-1.94)=0.0668-0.026= 0.0408[/tex]

Step-by-step explanation:

Let X the random variable that represent the temperatures of a population, and for this case we know the distribution for X is given by:

[tex]X \sim N(0,1)[/tex]  

Where [tex]\mu=0[/tex] and [tex]\sigma=1[/tex]

We are interested on this probability

[tex]P(-1.94<X<-1.5)[/tex]

And using the z score formula given by:

[tex]z=\frac{x-\mu}{\sigma}[/tex]

And using this formula we got:

[tex]P(-1.94<X<-1.5)=P(\frac{-1.94-\mu}{\sigma}<\frac{X-\mu}{\sigma}<\frac{-1.5-\mu}{\sigma})=P(\frac{-1.94-0}{1}<Z<\frac{-1.5-0}{1})=P(-1.94<z<-1.5)[/tex]

And we can find this probability with this difference:

[tex]P(-1.94<z<-1.5)=P(z<-1.5)-P(z<-1.94)=0.0668-0.026= 0.0408[/tex]

A fish tank has a base, B, with an area, in square inches, modeled by B(x) = 2x^2 + 6x + 4. The height, H, in inches, is modeled by H(x) = x + 3. Find the equation that models the fish tank’s volume, V, in cubic inches.

A. V(x) = 2x^2 + 7x + 7

B. V(x) = 2x^2 + 5x + 1

C. V(x) = 2x^3 + 12x^2 + 22x + 12

D. V(x) = 2x^3 + 8x^2 + 10x + 4

Answers

Answer:

C. V(x) = 2x^3 + 12x^2 + 22x + 12

Step-by-step explanation:

The volume of a fish tank is the multiplication of the area of the base by the height.

In this question:

Area of the base: [tex]B(x) = 2x^{2} + 6x + 4[/tex]

Height: [tex]H(x) = x + 3[/tex]

Volume:

[tex]V(x) = B(X)*H(x) = (2x^{2} + 6x + 4)(x + 3) = 2x^{3} + 6x^{2} + 4x + 6x^{2} + 18x + 12 = 2x^{3} + 12x^{2} + 22x + 12[/tex]

So the correct answer is:

C. V(x) = 2x^3 + 12x^2 + 22x + 12

Consider the following equation. f(x, y) = sin(4x + 3y), P(−6, 8), u = 1 2 3 i − j (a) Find the gradient of f. ∇f(x, y) = (b) Evaluate the gradient at the point P. ∇f(−6, 8) = (c) Find the rate of change of f at P in the direction of the vector u. Duf(−6, 8) =

Answers

Answer:

Step-by-step explanation:

a) Find the gradient

[tex]\nabla f(x,y) =f_x(x,y)\bold{i} +f_y (x,y)\bold{j}[/tex]

[tex]Also, sin(4x+3y)= sin(4x)cos(3y)+sin(3y)cos (4x)[/tex]

[tex]f_x(x,y) = 4cos(4x)cos(3y)-4sin(3y)sin(4x) = 4 cos(4x-3y)[/tex]

[tex]f_y(x,y) = -3sin(4x)sin(3y)+3cos(3y)cos(4x)=3cos(4x-3y)[/tex]

Hence

[tex]\nabla f(x,y) = 4cos(4x-3y)\bold{i}+3cos(4x-3y)\bold{j}[/tex]

b) At point (-6, 8) just replace the values in gradient to find it out. You can do it.

c) directional derivative in the direction u.

[tex]D_uf(x,y)=\nabla f(x,y). u[/tex]

I stuck with your 1 2 3 i -j . What does 1 2 3 mean? is it not 123 or 1 2/3 or else?

Pick 3 cards from a standard 52-card deck. Find the P(of at least 1 red card).

Does this experiment represent a binomial probability situation? Explain why or why not.

Answers

Answer:P(BBR) = 1/2 × 25/51 × 26/50 = 13/102 if cards are not replaced.

P(RBB) = 1/2 × 26/51 × 25/50 (simplified 1/2) = 13/102

P(BRB) = 1/2 × 26/51 ×25/50 (simplified 1/2) = 13/102

Step-by-step explanation: P(B) at first step is 26 cards out of a possible 52 therefore 26/52 (or simplified 1/2). We then have 25 black cards left out of a possible 51 therefore 25/51. The final card then has to be red to meet the criteria, we have 26 red cards still out of a possible 50 therefore 26/50.

This would be an example of binomial probability as at each step there are only 2 options R or B.

In one lottery, a player wins the jackpot by matching all five numbers drawn from white balls (1 through 41) and matching the number on the gold ball (1 through 31). If one ticket is purchased what is the probability of winning the jackpot?
(Type answer as an integer or a simplified fraction)

Answers

Answer:

1 / 2,787,760,560

Step-by-step explanation:

The number of permutations is:

41 × 40 × 39 × 38 × 37 × 31 = 2,787,760,560

So the probability of winning the jackpot is 1 / 2,787,760,560.

Suppose that you had consumer group wanted to test to see if weight of participants in a weight loss program changed (up or down). They computed a 95% confidence interval of the result (-4.977, 2.177). What do we know about the p-value for the test?

Answers

Answer:

What we know about the p-value for the test is that it will be less than 0.05

Step-by-step explanation:

The complete question is as follows;

Suppose that you had consumer group wanted to test to see if weight of participants in a weight loss program changed (up or down). They computed a 95% confidence interval of the result (-4.977, -2.177). Suppose that we had a significance test with the following hypothesis:

H0: population mean weight loss = 0

Ha: population mean weight loss does not equal 0

What do we know about the p-value for the test?

solution;

Given the information in the question, we are concerned with saying what we know about the p-value for the test

To answer this question, we shall be looking at it from the angle of the confidence interval

The confidence interval given is 95%.

Now what does this indicate?

It indicates that 95% confidence interval contains only values which are less than 0.

What does this mean for the p-value?

This means that the p-value will be less than 0.05

The Interplanet Space Centre is planning to launch a probe which will travel from Earth to Mars, on to Jupiter and then to Saturn. Distance from Earth to Mars 78 300 000 km Distance from Mars to Jupiter 550 400 000 km Distance from Jupiter to Saturn 648 700 000 km million km
Estimate to the nearest million how far it will travel on its journey.

Answers

Answer:

1,300,000,000

Step-by-step explanation:

 78    300   000

 550  400   000

 648  700   000

1  277  400   000

To the nearest million

1,2 | 77 400 000            more than 4, so +1

1,300,000,000

what does a shape need to be a prallelogram?

Answers

Answer: Needs to be flat with opposite sides that are parallel and are the same in length

What is the equation of the horizontal line through (-5,-2)

Answers

Answer:

  y = -2

Step-by-step explanation:

A horizontal line has an equation of the form ...

  y = constant

In order for the line to go through a point with a y-coordinate of -2, the constant must be -2.

  y = -2 . . . . horizontal line through (-5, -2)

What is the minimum and maximum whole number side length for △ABC with given side lengths of 9 cm and 11 cm? Please explain why

Answers

Answer:

Step-by-step explanation:

We would assume that triangle ABC is a right angled triangle. This means that we can apply Pythagoras theorem in determining the unknown side length.

For the case of the minimum side length, we would assume that the unknown length, L is one of the shorter legs of the triangle. By applying Pythagoras theorem, it becomes

11² = 9² + L²

L² = 121 - 81 = 40

L = √40 = 6.32

For the case of the maximum side length, we would assume that the unknown length, L is one of the hypotenuse of the triangle. By applying Pythagoras theorem, it becomes

L² = 9² + 11²

L² = 81 + 121 = 202

L = √202 = 14.21

The minimum side length is 6.32 and the maximum side length is 14.21

Would you rather 36 spherical bottles with radi = 4.57 or three 12-packs of cans if each box is 15inx5inx4.4in

Answers

Answer:

  36 bottles

Step-by-step explanation:

Each spherical bottle will have a volume of ...

  V = (4/3)πr³ = (4/3)π(4.57³) = 399.8 . . . . cubic inches

Each box (12-pack?) will have a volume of ...

  V = LWH = (15 in)(5 in)(4.4 in) = 330 in³

This is a smaller volume than even one spherical bottle, so 36 bottles will have the larger volume.

Which of the following measurements is the most precise?
A.A 15.5 foot tall tree
B.An 84-year-old man
C.A gas price of $2.359 per gallon

Answers

Answer:

C.)

Step-by-step explanation:

C.) Because it states the exact decimals rather than just stating about $2.36. Also, the other options were not specific. Hope this helps!  

I think it’s c. Because it’s the most precise going past three decimal places.

The football team makes a profit of 20% on all sales of jerseys. During one season, the total sales on jerseys were $2625. How much was the profit?

Answers

Answer:

2625*0.2= $525 in profit

Pls brainliest!!

There are 25 students in a class. Sixteen of those students are boys. What percent of the class are girls?

Answers

Answer:

36% of the students are girls

Step-by-step explanation:

(Boys) 16/25 = 64%

(Girls) 9/25 = 36%

Answer:

36%

Step-by-step explanation:

you first do 25 minus 16 to find out how many girls are in the class which is 9

now u do 9/25 = ?/100 to find out the percent of girls

9 times 100 divided by 25 which gives u 36%

hope this helps

Please answer this question and thank you !! Will give you brainliest !!

Answers

The answer is D(same slope and diff intercepts are the key to having parallel lines)

Answer:

D

Step-by-step explanation:

Two lines are parallel if they have the same slope

y= mx+b

those with same m,

It only appears in the last option where m= -5 for both lines, so option d

solve system by substitutition or elimination.
9x-8x=9
-18x+16y=-18

Answers

Answer:

1. 9x-8y=9

2. -18x+16y= -18 ⇒ -9x+8y= -9 ⇒ 9x-8y=9

as we see both equations are same, it means the lines overlap and there is infinite number of solutions

Solve for x.
6(x - 1) = 9(x + 2)
X = -8
X = -3
X = 3
x = 8

Answers

Answer:x=-8

Step-by-step explanation:


What is the quotient of 3/4 divided by 1/5

Answers

Answer:

15/4

Step-by-step explanation:

3/4:1/5= 3/4*5= 15/4

11. In your own words, describe how to view the
hean of a set of numerical data in two different
ways: in terms of leveling out and as a balance
point. In each case, give an example to illustrate.

Answers

Do they want me describe the step by step process or no???

A circle has a diameter of 10 feet. What is the area of the circle? Leave answers in terms of π.

Answers

Answer:

25pi

Step-by-step explanation:

If the circle has a diameter of 10, it has an radius of 5. The area of a circle is pi*r^2. If r=5, then it is 25pi.

Answer:

25π feet^2

Step-by-step explanation:

The area of a circle can be found using the following formula.

a=πr^2

We are given the diameter, but we need to find the radius. The radius is half of the diameter, or

r=d/2

We know the diameter is 10 feet. Therefore, we can substitute 10 in for d.

r=10/2

Divide 10 by 2

r=5

The radius is 5 feet.

Now we know the radius, and can substitute it into the area formula.

a=πr^2

r=5

a=π*5^2

Evaluate the exponent.

5^2 is equal to 5*5, which is 25.

a=π*25

a=25π

Add appropriate units. Area always uses units^2, and the units in this problem are feet.

a=25π feet^2

The area of the circle is 25π square feet

Which is the graph of the equation ? y-1=2/3(x-3)?

Answers

It’s the third graph.
Hope this helped:)

A function assigns values. The graph of the equation y-1=2/3(x-3) is the second graph.

What is a Function?

A function assigns the value of each element of one set to the other specific element of another set.

As the function is given to us that y-1=2/3(x-3), now substitute the value of x as 0 and y as 0 to get the intercept of the function. Therefore,

x-intercept

[tex]y-1=\dfrac23(x-3)\\0-1= \dfrac23x-\dfrac23(3)\\\\-1 = \dfrac23x - 2\\\\-1+2 = \dfrac23x\\\\x = \dfrac32[/tex]

y-intercept

[tex]y-1=\dfrac23(x-3)\\\\y-1=\dfrac23(0-3)\\\\y-1=\dfrac23(-3)\\\\y= -2+1\\\\y=-1[/tex]

Thus, we need to look for the graph which intersects the x-axis at 3/2 while the y-axis at -1. Therefore, as shown the below graph.

Hence, the graph of the equation y-1=2/3(x-3) is the second graph.

Learn more about Function:

https://brainly.com/question/5245372

What is x X/5=15. ⌚️⌚️⌚️⌚️⌚️⌚️⌚️⌚️⌚️⌚️⌚️⌚️⌚️⌚️⌚️⌚️⌚️⌚️⌚️⌚️⌚️⌚️⌚️⌚️⌚️⌚️⌚️⌚️⌚️

Answers

Answer:

x=3

Step-by-step explanation:

15/5=3 now I gotta get 20 chatacters

Answer:

[tex]x=75[/tex]

Step-by-step explanation:

[tex]\frac{x}{5} =15[/tex]

[tex]x=15 \times 5[/tex]

[tex]x=75[/tex]

Find the critical value needed to construct a confidence interval of the given level with the given sample size. Round the answers to three decimal places.
Part 1 of 4
(a) For level 95% and sample size 7.
Part 2 of 4
(b) For level 98% and sample size 12.
Part 3 of 4
(c) For level 99% and sample size 27.
Part 4 of 4
(d) For level 98% and sample size 14.

Answers

Answer:

a) [tex] t_{\alpha/2}=\pm 2.447[/tex]

b) [tex] t_{\alpha/2}=\pm 2.718[/tex]

c) [tex] t_{\alpha/2}=\pm 2.779[/tex]

d) [tex] t_{\alpha/2}=\pm 2.650[/tex]

Step-by-step explanation:

Part a

The degrees of freedom are given by:

[tex] df=n-1= 7-1=6[/tex]

The confidence is 95% or 0.95 the significance would be [tex]\alpha=0.05[/tex] and the critical values would be:

[tex] t_{\alpha/2}=\pm 2.447[/tex]

Part b

The degrees of freedom are given by:

[tex] df=n-1=12-1=11[/tex]

The confidence is 98% or 0.98 the significance would be [tex]\alpha=0.02[/tex] and the critical values would be:

[tex] t_{\alpha/2}=\pm 2.718[/tex]

Part c

The degrees of freedom are given by:

[tex] df=n-1=27-1=26[/tex]

The confidence is 99% or 0.99 the significance would be [tex]\alpha=0.01[/tex] and the critical values would be:

[tex] t_{\alpha/2}=\pm 2.779[/tex]

Part d

The degrees of freedom are given by:

[tex] df=n-1=14-1=13[/tex]

The confidence is 98% or 0.98 the significance would be [tex]\alpha=0.02[/tex] and the critical values would be:

[tex] t_{\alpha/2}=\pm 2.650[/tex]

A value of a test statistic defines the upper bounds of a confidence interval, statistical significance in a test statistic is known as the crucial value.

critical value:

For part a )

n = 7

Calculating the degrees of freedom [tex]= df = n - 1 = 7 - 1 = 6[/tex]

At [tex]95\%[/tex] the confidence level, the t:

[tex]\alpha = 1 - 95\% = 1 - 0.95 = 0.05\\\\\frac{\alpha}{ 2} = \frac{0.05}{ 2} = 0.025\\\\t_{\frac{\alpha}{ 2}}\\\\df = t_{0.025,6} = 2.447\\\\[/tex]

The critical value = 2.447

For part b )

n =12

Calculating the degrees of freedom[tex]= df = n - 1 = 12 - 1 = 11[/tex]

At [tex]98\%[/tex] the confidence level , the t:

[tex]\alpha = 1 - 98\% = 1 - 0.98 = 0.02\\\\\frac{\alpha}{ 2}= \frac{0.02}{ 2} = 0.01\\\\t_{\frac{\alpha}{ 2}}\\\\df = t_{0.01,11} =2.718\\\\[/tex]

The critical value =2.718

For part c )

n = 27

Calculating the degrees of freedom [tex]= df = n - 1 = 27 - 1 = 26[/tex]

At [tex]99\%[/tex] the confidence level, the t:

[tex]\alpha = 1 - 99\% = 1 - 0.99 = 0.01\\\\\frac{\alpha}{ 2} = \frac{0.01}{ 2} = 0.005\\\\t_{\frac{\alpha}{ 2}}\\\\df = t_{0.005,26}=2.779\\\\[/tex]

The critical value =2.779

For part d )

n = 14

Calculating the degrees of freedom [tex]= df = n - 1 = 14 - 1 = 13[/tex]

At [tex]98\%[/tex] the confidence level, the t:

[tex]\alpha = 1 - 98\% = 1 - 0.98 = 0.02\\\\\frac{\alpha}{ 2} = \frac{0.02}{ 2} = 0.01\\\\t_{\frac{\alpha}{ 2}} \\\\df = t_{0.01,13} =2.650[/tex]

The critical value =2.650

Find out more about the critical value here:

brainly.com/question/14508634

5 customers buy 10,15,20,25,30 toffees respectively from 1 shop. But the the person who bought 20 toffees forgot to pay .if the shopkeeper makes 20% profit what will be his profit or loss from these five customers of the person who forgot to pay at pay his amount had paid his amount?
50% profit ,60% profit, 40% loss ,20% loss.

Answers

Answer:

40% profit

Step by step Explanation:

Profit percentage

=( profit/cost price) * 100

0.2 = profit/cost

10+15+20+25+30= 100

Let's assume the cost price of the items is $1 each

Cost price total= $100

Profit made when buyer of 20 toffe didn't say was

0.2=profit/cost

0.2*100 =$20

If the$ 20 paid.

Total profit = $40

So percentage profit now

40/100 * 100 = 40%

The shopkeeper makes a loss of 40% and this can be determined by using the given data.

Given :

5 customers buy 10,15,20,25,30 toffees respectively from 1 shop. But the person who bought 20 toffees forgot to pay.The shopkeeper makes a 20% profit.

The following steps can be used in order to determine the profit or loss the shopkeeper makes:

Step 1 - The total number of toffees can be calculated as:

[tex]\rm Total \; Toffees = 10 +15 + 20+ 25+ 30\\\rm Total \; Toffees = 100\\[/tex]

Step 2 - Now, let the cost of 1 toffee be $1. So, the cost of 80 toffees is $80.

Step 3 - So, the value of 20% profit by selling 80 toffees be 'x'. So, the value of 'x' is:

[tex]x = \dfrac{20}{100}\times 80[/tex]

[tex]x = \$16[/tex]

Step 4 - Now, let the profit or loss be 'y'. So, the value of 'y' is:

[tex]y = \dfrac{20}{80}\times 16[/tex]

[tex]y = \$4[/tex]

Step 5 - So, the loss percentage is 40%.

Therefore, the correct option is C).

For more information, refer to the link given below:

https://brainly.com/question/1494270

What’s the correct answer for this?

Answers

Answer:

6

Explanation:

According to secant-secant theorem,

(PB)(PA)=(PD)(PC)

(7)(12)=(PD)(14)

NOW

84/14 = PD

PD = 6

Other Questions
How to calculate the height of an object using trigonometry ASAP! GIVING BRAINLIEST! Please read the question THEN answer CORRECTLY! NO guessing. I say no guessing because people usually guess on my questions. How the growth of railroads transformed america during the nineteenth century? Read Shakespeares blow blow thou winter wind how does the use of poetic devices further the poems theme Please answer the attachment below correctly! Choose the sentence that reflects a writer's meta awareness. A.Betty often uses lively dialogue in her essays. B.Neil surveys the audience regarding their likes and dislikes after his speech. C.Josh never changes his writing style because his voice is so unique. D.Carmen considers her rhetorical situation before she begins writing. Type the expression that results from the following series of steps:Start with t times by 5. What is the slope of a line between the points (-4,-3) and (4, -2)? Migration isa. the movement of organisms from a native location to a foreign locationb. the movement of organisms from a foreign location to a native locationc. the movement of organisms from their water supply to their food supplyd. the seasonal movement of organisms between locations In which of the following US cities were major immigration stations located? * Required1What is an disadvantage of asexual reproduction?1.An organism can reproduce without a mate.2.There is little genetic variation within a population.3.Some organisms can quickly produce a large number of offspring. needed asap importance of e commerce Urgent. Please help. I need the correct answer f(x)=4x^+1 and g(x)=x^-5 find (f+g)(x) Emily has a bag of 20 fruit flavoured sweets 7 of the sweets are strawberry flavoured 11 are lime flavour and two are lemon flavour Emily takes a random sweet from the bag write down the probability that emily took the orange flavour You are conducting a study to see if the proportion of men over the age of 50 who regularly have their prostate examined is significantly less than 0.37. A random sample of 780 men over the age of 50 found that 206 have their prostate regularly examined. Do the sample data provide convincing evidence to support the claim. Test the relevant hypotheses using a 10% level of significance. Give answers to at least 4 decimal places. What are the correct hypotheses? Ari is arranging her coin collection. She has 11 pennies, 5 nickels, 8 dimes, 7 quarters, and 9 half-dollars. If she chooses one at random, what is the probability that it is a penny? Kayak Co. budgeted the following cash receipts (excluding cash receipts from loans received) and cash payments (excluding cash payments for loan principal and interest payments) for the first three months of next year. CashReceipts CashpaymentsJanuary$525,000 $475,000 February 400,000 350,000 March 450,000 525,000 According to a credit agreement with its bank, Kayak requires a minimum cash balance of $30,000 at each month-end. In return, the bank has agreed that the company can borrow up to $150,000 at a monthly interest rate of 1%, paid on the last day of each month. The interest is computed based on the beginning balance of the loan for the month. The company repays loan principal with any cash in excess of $30,000 on the last day of each month. The company has a cash balance of $30,000 and a loan balance of $60,000 at January 1.Prepare monthly cash budgets for each of the first three months of next year. Describe any events that have affected your relationship recently. in which country tries was islam established through traded which countries was islam established by invading islamic armies?